Những câu hỏi liên quan
Nguyen Khanh Huyen
Xem chi tiết
dbrby
Xem chi tiết
Nguyễn Việt Lâm
2 tháng 11 2019 lúc 22:48

\(A\ge7\left(a+b+c\right)^2+12\left(a+b+c\right)^2+\frac{18135}{a+b+c}\)

Đặt \(a+b+c=x\Rightarrow0< x\le2\)

\(A\ge19x^2+\frac{18135}{x}=19x^2+\frac{152}{x}+\frac{152}{x}+\frac{17831}{x}\)

\(A\ge3\sqrt[3]{\frac{19.152.152x^2}{x^2}}+\frac{17831}{2}=\frac{18287}{2}\)

Bình luận (0)
 Khách vãng lai đã xóa
l҉o҉n҉g҉ d҉z҉
Xem chi tiết
alibaba nguyễn
31 tháng 3 2021 lúc 13:53

Đề phải là số thực không âm mới đúng

Bình luận (0)
 Khách vãng lai đã xóa
WTFシSnow
Xem chi tiết
WTFシSnow
11 tháng 4 2021 lúc 22:01

alibaba nguyễn giúp em với WTFシSnow WTFシSnow 

Bình luận (0)
 Khách vãng lai đã xóa
kaneki_ken
Xem chi tiết
Nguyễn Thiều Công Thành
Xem chi tiết
Lầy Văn Lội
13 tháng 8 2017 lúc 22:33

\(P=\frac{a^3}{\left(a+1\right)\left(b+1\right)}+\frac{b^3}{\left(b+1\right)\left(c+1\right)}+\frac{c^3}{\left(c+1\right)\left(a+1\right)}-1\)

Bình luận (0)
Bá đạo sever là tao
13 tháng 8 2017 lúc 22:36

ôi trá hình :VVV

Bình luận (0)
tth_new
14 tháng 8 2017 lúc 20:18

\(P=\frac{a^3}{\left(a+1\right).\left(b+1\right)}+\frac{b^3}{\left(b+1\right).\left(c+1\right)}+\frac{c^3}{\left(c+1\right).\left(a+1\right)}\)

Ko biết đúng hay không!

Mới lớp 6 , mà tôi nghĩ Lầy Văn Lội đúng đấy!

Bình luận (0)
Nguyễn Thiều Công Thành
Xem chi tiết
Thắng Nguyễn
8 tháng 8 2017 lúc 17:18

bài này mà giải theo SOS là hơi bị tuyệt vời nhé =)))

Bình luận (0)
tuan pham anh
8 tháng 8 2017 lúc 17:21

em moi co lop 7

Bình luận (0)
Đỗ Thị Mai Anh
8 tháng 8 2017 lúc 20:34

em mới có lớp 6 thôi mà

Bình luận (0)
Le Trang Nhung
Xem chi tiết
Trần Hữu Ngọc Minh
7 tháng 12 2017 lúc 16:56

bài 1

ÁP dụng AM-GM ta có:

\(\frac{a^3}{b\left(2c+a\right)}+\frac{2c+a}{9}+\frac{b}{3}\ge3\sqrt[3]{\frac{a^3.\left(2c+a\right).b}{b\left(2c+a\right).27}}=a.\)

tương tự ta có:\(\frac{b^3}{c\left(2a+b\right)}+\frac{2a+b}{9}+\frac{c}{3}\ge b,\frac{c^3}{a\left(2b+c\right)}+\frac{2b+c}{9}+\frac{a}{3}\ge c\)

công tất cả lại ta có:

\(P+\frac{2a+b}{9}+\frac{2b+c}{9}+\frac{2c+a}{9}+\frac{a+b+c}{3}\ge a+b+c\)

\(P+\frac{2\left(a+b+c\right)}{3}\ge a+b+c\)

Thay \(a+b+c=3\)vào ta được":

\(P+2\ge3\Leftrightarrow P\ge1\)

Vậy Min là \(1\)

dấu \(=\)xảy ra khi \(a=b=c=1\)

Bình luận (0)
Hắc Dương
Xem chi tiết
Thiên An
22 tháng 6 2017 lúc 20:39

Kurosaki Akatsu giải thế thì đề bài cho  \(b^2+c^2\le a^2\)  để làm gì?

Bình luận (0)
Kurosaki Akatsu
22 tháng 6 2017 lúc 19:59

Áp dụng bất đẳng thức AM-GM ta có :

\(P=\frac{1}{a^2}\left(b^2+c^2\right)+a^2\left(\frac{1}{b^2}+\frac{1}{c^2}\right)\)

\(P=\frac{b^2}{a^2}+\frac{c^2}{a^2}+\frac{a^2}{b^2}+\frac{a^2}{c^2}\ge4.\sqrt[4]{\frac{b^2}{a^2}.\frac{c^2}{a^2}.\frac{a^2}{b^2}.\frac{a^2}{c^2}}=4.1=4\)

=> \(Min_P=4\)

Bình luận (0)
Thiên An
24 tháng 6 2017 lúc 21:51

Với a, b, c thực dương áp dụng BĐT Cô-si ta có:

\(P=\frac{1}{a^2}\left(b^2+c^2\right)+a^2\left(\frac{1}{b^2}+\frac{1}{c^2}\right)=\left(\frac{b^2}{a^2}+\frac{c^2}{a^2}\right)+\left(\frac{a^2}{b^2}+\frac{a^2}{c^2}\right)\)

\(\ge2\sqrt{\frac{b^2}{a^2}.\frac{c^2}{a^2}}+2\sqrt{\frac{a^2}{b^2}.\frac{a^2}{c^2}}=2\left(\frac{bc}{a^2}+\frac{a^2}{bc}\right)\)

\(=2\left[\left(\frac{bc}{a^2}+\frac{a^2}{4bc}\right)+\frac{3a^2}{4bc}\right]\ge2\left(2.\sqrt{\frac{bc}{a^2}.\frac{a^2}{4bc}}+\frac{3\left(b^2+c^2\right)}{4bc}\right)\)   (vì  \(a^2\ge b^2+c^2\))

\(=2\left(2\sqrt{\frac{1}{4}}+\frac{3.2bc}{4bc}\right)\) (vì  \(b^2+c^2\ge2bc\))

\(=2\left(2.\frac{1}{2}+\frac{3}{2}\right)=5\)

Vậy  Pmin = 5

Đẳng thức xảy ra  \(\Leftrightarrow\)  \(\hept{\begin{cases}a^2=b^2+c^2\\b=c\end{cases}}\)

Bình luận (0)